Podstawy statystyki: średnia, dominanta, kwartyl, wariancja

Procesy stochastyczne. Sposoby racjonalizowania wielkich ilości informacji. Matematyka w naukach społecznych.
RagnarPL
Użytkownik
Użytkownik
Posty: 3
Rejestracja: 10 gru 2017, o 14:07
Płeć: Mężczyzna
Lokalizacja: Kraków
Podziękował: 1 raz

Podstawy statystyki: średnia, dominanta, kwartyl, wariancja

Post autor: RagnarPL »

Do sprawdzenia:

Zad. 1. W klasie IV szkoły podstawowej w pewnej miejscowości dokonano pomiaru poziomu inteligencji uczniów. Wyniki tych pomiarów (w punktach) wraz z oznaczeniami płci są następujące: 127(D), 129(D), 130(C), 132(C), 134(C), 135(D), 135(C), 135(C), 135(C), 140(C), 142(D), 143(D), 143(C), 144(C), 145(D), 145(D), 149(D), 152(C), 152(D), 153(D). Przeanalizuj szereg przedstawiający poziom inteligencji w tej klasie. W tym celu wyznacz wartość: średniej arytmetycznej, dominantę (modalną), medianę, kwartyl dolny i górny, rozstępu, wariancji, odchylenie standardowe oraz współczynnik zmienności. Dokonaj analizy porównawczej poziomu inteligencji dzieci, dzieląc całą ich zbiorowość na dwie populacje: dziewcząt i chłopców.

Dla chłopców:

\(\displaystyle{ \begin{tabular}{c|c|c|c|c|c|c|c|c}
x_{i} & 130 & 132 & 134 & 135 & 140 & 143 & 144 & 152\\
\hline
n_{i} & 1 & 1 & 1 & 3 & 1 & 1 & 1 & 1\\
\end{tabular}}\)


a) średnia arytmetyczna: \(\displaystyle{ \overline{x} = \frac{130+132+134+3*135+140+143+144+152}{10} = \frac{1380}{10} = 138}\)

b) dominanta (modalna): \(\displaystyle{ D = 135}\)

c) mediana (n parzyste): \(\displaystyle{ Me = \frac{1}{2}(135 + 135) = 135}\)

d) kwartyl dolny: \(\displaystyle{ \frac{n}{4} = \frac{10}{4} = 2,5}\) ---------- \(\displaystyle{ Q_{1} = 134+\frac{2,5-2}{1}*0 = 134}\)

e) kwartyl górny: \(\displaystyle{ \frac{3n}{4} = \frac{30}{4} = 7,5}\) ---------- \(\displaystyle{ Q_{3} = 143+\frac{7,5-7}{1}*0 = 143}\)

f) rozstęp: \(\displaystyle{ 152 - 130 = 22}\)

g) wariancja: \(\displaystyle{ \sigma^{2} = \frac{(-8)^{2}+(-6)^{2}+(-4)^{2}+3*(-3)^{2}+2^{2}+5^{2}+6^{2}+14^{2}}{10} = \frac{404}{10} = 40,4}\)

h) odchylenie standardowe: \(\displaystyle{ \sigma = \sqrt{40,4} \approx 6,3561}\)

i) współczynnik zmienności: \(\displaystyle{ V = \frac{6,3561}{138}*100 \% \approx 4,61 \%}\)

Dla dziewczyn tak samo.

-- 12 gru 2017, o 15:30 --

Do sprawdzenia:

Zad. 2. W urnie jest pięć kul ponumerowanych od 1 do 5. Z tej urny losujemy jednocześnie dwie kule. Niech X będzie sumą numerów dwóch wylosowanych kul. Z jakim prawdopodobieństwem zmienna losowa X przyjmuje wartość równą co najmniej 6? Wyznacz dominantę, medianę, wartość oczekiwaną oraz odchylenie standardowe zmiennej losowej X.

\(\displaystyle{ \Omega = 10}\)
\(\displaystyle{ \Omega = \{(1,2), (1,3), (1,4), (1,5), (2,3), (2,4), (2,5), (3,4), (3,5), (4,5)\}}\)

\(\displaystyle{ \begin{tabular}{c|c|c|c|c|c|c|c}
x_{i} & 3 & 4 & 5 & 6 & 7 & 8 & 9\\
\hline
n_{i} & 1 & 1 & 2 & 2 & 2 & 1 & 1\\
\end{tabular}}\)


Rozkład prawdopodobieństwa:
\(\displaystyle{ \begin{tabular}{c|c|c|c|c|c|c|c}
x_{i} & 3 & 4 & 5 & 6 & 7 & 8 & 9\\
\hline
p_{i} & \(\frac{1}{10} & \(\frac{1}{10} & \(\frac{2}{10} & \(\frac{2}{10} & \(\frac{2}{10} & \(\frac{1}{10} & \(\frac{1}{10}\\
\end{tabular}}\)


a) zmienna losowa X przyjmuje wartość równą co najmniej 6 z prawdopodobieństwem: \(\displaystyle{ P(X \ge 6)=\frac{6}{10}}\)

b) dominanta: \(\displaystyle{ D = 5, 6, 7}\)

c) mediana: Me = 6
\(\displaystyle{ P(X \le 6) = \frac{1}{10}+\frac{1}{10}+\frac{2}{10}+\frac{2}{10} = \frac{6}{10} \ge \frac{1}{2}}\)
\(\displaystyle{ P(X \ge 6) = \frac{1}{10}+\frac{1}{10}+\frac{2}{10}+\frac{2}{10} = \frac{6}{10} \ge \frac{1}{2}}\)

d) wartość oczekiwana: \(\displaystyle{ E(X) = 3*0,1+4*0,1+5*0,2+6*0,2+7*0,2+8*0,1+9*0,1 = 6}\)

e) odchylenie standardowe: \(\displaystyle{ D(X) = \sqrt{D^{2}(X)} = \sqrt{(-3)^{2}*0,1+(-2)^{2}*0,1+(-1)^{2}*0,2+0^{2}*0,2+1^{2}*0,2+2^{2}*0,1+3^{2}*0,1} = \sqrt3 \approx 1,73}\)

-- 12 gru 2017, o 15:35 --

Do sprawdzenia:

Zad. 3. W urnie jest 5 kul białych i 1 czarna. Losujemy bez zwracania kulę z tej urny tak długo, aż wylosujemy kulę czarną. Określ rozkład czasu trwania takiego doświadczenia losowego.

\(\displaystyle{ \Omega = 6}\)
\(\displaystyle{ \Omega = \{c, bc, bbc, bbbc, bbbbc, bbbbbc\}}\)

Rozkład prawdopodobieństwa:
\(\displaystyle{ \begin{tabular}{c|c|c|c|c|c|c}
x_{i} & 1 & 2 & 3 & 4 & 5 & 6\\
\hline
p_{i} & \(\frac{1}{6} & \(\frac{1}{6} & \(\frac{1}{6} & \(\frac{1}{6} & \(\frac{1}{6} & \(\frac{1}{6}\\
\end{tabular}}\)


- dominanta: \(\displaystyle{ D = 1, 2, 3, 4, 5, 6}\)

- mediana: Me = <3, 4>
\(\displaystyle{ P(X \le 3) = \frac{1}{6}+\frac{1}{6}+\frac{1}{6} = \frac{3}{6} \ge \frac{1}{2}}\)
\(\displaystyle{ P(X \ge 3) = \frac{1}{6}+\frac{1}{6}+\frac{1}{6} = \frac{3}{6} \ge \frac{1}{2}}\)
\(\displaystyle{ P(X \le 4) = \frac{1}{6}+\frac{1}{6}+\frac{1}{6} = \frac{3}{6} \ge \frac{1}{2}}\)
\(\displaystyle{ P(X \ge 4) = \frac{1}{6}+\frac{1}{6}+\frac{1}{6} = \frac{3}{6} \ge \frac{1}{2}}\)

- wartość oczekiwana: \(\displaystyle{ E(X) = 1*\frac{1}{6}+2*\frac{1}{6}+3*\frac{1}{6}+4*\frac{1}{6}+5*\frac{1}{6}+6*\frac{1}{6} = 3\frac{1}{2}}\)

- wariacja:
\(\displaystyle{ D^{2}(X) = (-2\frac{1}{2})^{2}*\frac{1}{6}+(-1\frac{1}{2})^{2}*\frac{1}{6}+ (-\frac{1}{2})^{2}*\frac{1}{6}+(\frac{1}{2})^{2}*\frac{1}{6}+(1\frac{1}{2})^{2}*\frac{1}{6}+(2\frac{1}{2})^{2}*\frac{1}{6} = \frac{70}{24} \approx 2,92}\)

- odchylenie standardowe: \(\displaystyle{ D(X) = \sqrt{2,92} \approx 1,71}\)

Do sprawdzenia:

Zad. 4. Rzucamy monetą tak długo, aż wyrzucimy reszkę, ale nie więcej niż 6 razy. Liczba wykonanych rzutów jest zmienną losową T. Jest ona czasem trwania doświadczenia losowego. Określ jej rozkład.

\(\displaystyle{ \Omega = 6}\)
\(\displaystyle{ \Omega = \{r, or, oor, ooor, oooor, ooooor, oooooo\}}\)

Rozkład prawdopodobieństwa:
\(\displaystyle{ \begin{tabular}{c|c|c|c|c|c|c|c}
x_{i} & 1 & 2 & 3 & 4 & 5 & 6 & 7\\
\hline
p_{i} & \(\frac{1}{2} & \(\frac{1}{4} & \(\frac{1}{8} & \(\frac{1}{16} & \(\frac{1}{32} & \(\frac{1}{64} & \(\frac{1}{64}\\
\end{tabular}}\)


- dominanta: \(\displaystyle{ D = 1}\)

- mediana: Me = <1, 2>
\(\displaystyle{ P(X \le 1) = \frac{1}{2}\ge \frac{1}{2}}\)
\(\displaystyle{ P(X \ge 1) = \frac{1}{64}+\frac{1}{64}+\frac{1}{32}+\frac{1}{16} + \frac{1}{8} + \frac{1}{4} + \frac{1}{2} = 1 \ge \frac{1}{2}}\)
\(\displaystyle{ P(X \le 2) = \frac{1}{2}+\frac{1}{4} = \frac{3}{4} \ge \frac{1}{2}}\)
\(\displaystyle{ P(X \ge 2) = \frac{1}{64}+\frac{1}{64}+\frac{1}{32}+\frac{1}{16} + \frac{1}{8} + \frac{1}{4} = \frac{1}{2} \ge \frac{1}{2}}\)

- wartość oczekiwana: \(\displaystyle{ E(X) = 1*\frac{1}{2}+2*\frac{1}{4}+3*\frac{1}{8}+4*\frac{1}{16}+5*\frac{1}{32}+6*\frac{1}{64}+7*\frac{1}{64} = 1\frac{47}{64}}\)

Do sprawdzenia:

Zad. 5. Rzucono cztery razy symetryczną monetą. Niech zdarzenie X oznacza liczbę wyrzuconych reszek. Należy: zbudować rozkład prawdopodobieństwa zmiennej losowej X, określić dystrybuantę tej zmiennej losowej i narysować jej wykres, znaleźć wartość parametrów: E(X), D2(X) oraz D(X) i obliczyć prawdopodobieństwo uzyskania co najmniej raz reszki.
(Schemat Bernoullego) \(\displaystyle{ P(k) = \binom{n}{k}*p^{k}*q^{n-k}}\)
n - liczba rzutów, k - liczba sukcesów, p -prawdopodobieństwo sukcesu, q - prawdopodobieństwo porażki

\(\displaystyle{ n = 4, p = \frac{1}{2}, q = \frac{1}{2}}\)

\(\displaystyle{ P(X=0) = \binom{4}{0}*(\frac{1}{2})^{0}*(\frac{1}{2})^{4}=1*1*\frac{1}{16} = \frac{1}{16}}\)
\(\displaystyle{ P(X=1) = \binom{4}{1}*(\frac{1}{2})^{1}*(\frac{1}{2})^{3}=4*\frac{1}{2}*\frac{1}{8} = \frac{4}{16}}\)
\(\displaystyle{ P(X=2) = \binom{4}{2}*(\frac{1}{2})^{2}*(\frac{1}{2})^{2}=6*\frac{1}{4}*\frac{1}{4} = \frac{6}{16}}\)
\(\displaystyle{ P(X=3) = \binom{4}{3}*(\frac{1}{2})^{3}*(\frac{1}{2})^{1}=4*\frac{1}{8}*\frac{1}{2} = \frac{4}{16}}\)
\(\displaystyle{ P(X=4) = \binom{4}{4}*(\frac{1}{2})^{4}*(\frac{1}{2})^{0}=1*\frac{1}{16}*1 = \frac{1}{16}}\)

Rozkład prawdopodobieństwa:
\(\displaystyle{ \begin{tabular}{c|c|c|c|c|c}
x_{i} & 0 & 1 & 2 & 3 & 4\\
\hline
p_{i} & \(\frac{1}{16} & \(\frac{4}{16} & \(\frac{6}{16} & \(\frac{4}{16} & \(\frac{1}{16}\\
\end{tabular}}\)


- wartość oczekiwana: \(\displaystyle{ E(X) = 0*\frac{1}{16}+1*\frac{4}{16}+2*\frac{6}{16}+3*\frac{4}{16}+4*\frac{1}{16} = 2}\)

- wariacja:
\(\displaystyle{ D^{2}(X) = (-2)^{2}*\frac{1}{16}+(-1)^{2}*\frac{4}{16}+ 0^{2}*\frac{6}{16}+1^{2}*\frac{4}{16}+2^{2}*\frac{1}{16} = 1}\)

- odchylenie standardowe: \(\displaystyle{ D(X) = \sqrt{1} = 1}\)

- prawdopodobieństwo uzyskania co najmniej raz reszki: \(\displaystyle{ P(X \ge 1) = \frac{15}{16}}\)

- dystrybuanta: \(\displaystyle{ F(x) = P(X<x)}\)

\(\displaystyle{ F(x) = \begin{cases} 0 &\text{dla } x \epsilon(-\infty, 0> \\\frac{1}{16} &\text{dla } x\epsilon(0, 1>\\\frac{5}{16} &\text{dla } x\epsilon(1, 2>\\\frac{11}{16} &\text{dla } x\epsilon(2, 3>\\\frac{15}{16} &\text{dla } x\epsilon(3, 4>\\1 &\text{dla } x\epsilon(4, \infty) \end{cases}}\)

- rysunek


Do sprawdzenia:

Zad. 6. Rzucono cztery razy symetryczną kostką do gry. Niech zmienną losową X będzie liczba otrzymanych wyników podzielnych przez trzy. Należy: znaleźć rozkład zmiennej losowej X, zapisać funkcje dystrybuanty tej zmiennej losowej, znaleźć E(X), D2(X) i D(X)i porównać otrzymany rozkład z rozkładem z poprzedniego zadania.

\(\displaystyle{ n = 4, p = \frac{2}{6} = \frac{1}{3}, q = \frac{4}{6} = \frac{2}{3}}\)

\(\displaystyle{ P(X=0) = \binom{4}{0}*(\frac{1}{3})^{0}*(\frac{2}{3})^{4}=1*1*\frac{16}{81} = \frac{16}{81}}\)
\(\displaystyle{ P(X=1) = \binom{4}{1}*(\frac{1}{3})^{1}*(\frac{2}{3})^{3}=4*\frac{1}{3}*\frac{8}{27} = \frac{32}{81}}\)
\(\displaystyle{ P(X=2) = \binom{4}{2}*(\frac{1}{3})^{2}*(\frac{2}{3})^{2}=6*\frac{1}{9}*\frac{4}{9} = \frac{24}{81}}\)
\(\displaystyle{ P(X=3) = \binom{4}{3}*(\frac{1}{3})^{3}*(\frac{2}{3})^{1}=4*\frac{1}{27}*\frac{2}{3} = \frac{8}{81}}\)
\(\displaystyle{ P(X=4) = \binom{4}{4}*(\frac{1}{3})^{4}*(\frac{2}{3})^{0}=1*\frac{1}{81}*1 = \frac{1}{81}}\)

Rozkład prawdopodobieństwa:
\(\displaystyle{ \begin{tabular}{c|c|c|c|c|c}
x_{i} & 0 & 1 & 2 & 3 & 4\\
\hline
p_{i} & \(\frac{16}{81} & \(\frac{32}{81} & \(\frac{24}{81} & \(\frac{8}{81} & \(\frac{1}{81}\\
\end{tabular}}\)


- wartość oczekiwana (tylko Schemat Bernoulliego): \(\displaystyle{ n*p}\)
\(\displaystyle{ E(X) = 4*\frac{1}{3} = 1\frac{1}{3}}\)

- wariacja (tylko Schemat Bernoulliego): \(\displaystyle{ n*p(1-p)}\):
\(\displaystyle{ D^{2}(X) = 4*\frac{1}{3}(1-\frac{1}{3}) = \frac{4}{3}*\frac{2}{3} = \frac{8}{9}}\)

- odchylenie standardowe: \(\displaystyle{ D(X) = \sqrt{\frac{8}{9}} = \frac{2\sqrt2}{3}}\)

- dystrybuanta: \(\displaystyle{ F(x) = P(X<x)}\)

\(\displaystyle{ F(x) = \begin{cases} 0 &\text{dla } x \epsilon(-\infty, 0> \\\frac{16}{81} &\text{dla } x\epsilon(0, 1>\\\frac{48}{81} &\text{dla } x\epsilon(1, 2>\\\frac{72}{81} &\text{dla } x\epsilon(2, 3>\\\frac{80}{81} &\text{dla } x\epsilon(3, 4>\\1 &\text{dla } x\epsilon(4, \infty) \end{cases}}\)

Nie mam pojęcia jak to wgl. zacząć. Jakieś podpowiedzi?

Zad. 7. Na każde spośród 10 pytań wybieramy losowo jedną z 5 odpowiedzi. Zmienna losowa X przyporządkowuje każdemu wynikowi ilość poprawnych odpowiedzi. Wyznacz najmniejsze k tak, aby P(X >= k) < 0,05.

Do sprawdzenia:

Zad. 8. Rozważmy funkcję \(\displaystyle{ f(x) = \begin{cases} 0 &\text{dla } x \epsilon(-\infty, 0> \cup (\sqrt2, +\infty) \\x &\text{dla } x\epsilon(0, \sqrt2>\end{cases}}\) Należy: wykazać, że jest funkcją gęstości pewnej zmiennej losowej X, wyznaczyć jej dystrybuantę i obliczyć prawdopodobieństwo, że zmienna losowa \(\displaystyle{ X<1, X=1,4, X\epsilon\left\langle \frac{3}{5}, \frac{6}{5}\right\rangle}\).

\(\displaystyle{ \int\limits_{-\infty}^{+\infty}f(x)dx = \int\limits_{-\infty}^{0}0dx + \int\limits_{0}^{\sqrt2}xdx + \int\limits_{\sqrt2}^{+\infty}0dx = \frac{1}{2}x^{2}\begin{tabular}{|rc|c}
\sqrt2\\
\\
0\\
\end{tabular}}\)
\(\displaystyle{ = \frac{1}{2}*{(\sqrt2)}^{2}-\frac{1}{2}*0^{2} = 1}\)

- ta funkcja jest funkcją gęstości pewnej zmiennej losowej X: \(\displaystyle{ \int\limits_{-\infty}^{+\infty}f(x)dx = 1}\)
- dystrybuanta:

\(\displaystyle{ dla...x\epsilon(-\infty, 0>}\)

\(\displaystyle{ \int\limits_{-\infty}^{x}f(t)dt = \int\limits_{-\infty}^{x}0dt = 0}\)

\(\displaystyle{ dla...x\epsilon(0, \sqrt2>}\)

\(\displaystyle{ \int\limits_{-\infty}^{x}f(t)dt = \int\limits_{-\infty}^{0}0dt + \int\limits_{0}^{x}tdt = \frac{1}{2}t^{2}\begin{tabular}{|rc|c}
x\\
\\
0\\
\end{tabular}}\)
\(\displaystyle{ =\frac{1}{2}x^{2}-\frac{1}{2}*0^{2} = \frac{1}{2}x^{2}}\)

\(\displaystyle{ dla...x\epsilon(\sqrt2, +\infty)}\)

\(\displaystyle{ \int\limits_{-\infty}^{x}f(t)dt = \int\limits_{-\infty}^{0}0dt + \int\limits_{0}^{\sqrt2}tdt +
\int\limits_{\sqrt2}^{x}0dt = \frac{1}{2}t^{2}\begin{tabular}{|rc|c}
\sqrt2\\
\\
0\\
\end{tabular}}\)
\(\displaystyle{ = \frac{1}{2}*{(\sqrt2)}^{2}-\frac{1}{2}*0^{2} = 1}\)

\(\displaystyle{ F(x) = \begin{cases} 0 &\text{dla } x \epsilon(-\infty, 0> \\\frac{1}{2}x^{2} &\text{dla } x\epsilon(0, \sqrt2>\\1 &\text{dla } x\epsilon(\sqrt2, +\infty>\\ \end{cases}}\)

- prawdopodobieństwo:

\(\displaystyle{ P(X<1) = \int\limits_{-\infty}^{1}f(x)dx = \int\limits_{-\infty}^{0}0dx + \int\limits_{0}^{1}xdx = \frac{1}{2}x^{2}\begin{tabular}{|rc|c}
1\\
\\
0\\
\end{tabular}}\)
\(\displaystyle{ = \frac{1}{2}*1^{2}-\frac{1}{2}*0^{2} = \frac{1}{2}}\)

\(\displaystyle{ P(X=1,4) = 0}\) - zawsze?

\(\displaystyle{ P(X\epsilon\left\langle \frac{3}{5}, \frac{6}{5}\right\rangle) = \int\limits_{\frac{3}{5}}^{\frac{6}{5}}f(x)dx = \int\limits_{\frac{3}{5}}^{\frac{6}{5}}xdx = \frac{1}{2}x^{2}\begin{tabular}{|rc|c}
\(\frac{6}{5}\\
\\
\(\frac{3}{5}\\
\end{tabular}}\)
\(\displaystyle{ = \frac{1}{2}*{(\frac{6}{5})}^{2}-\frac{1}{2}*{(\frac{3}{5})}^{2} = \frac{36}{50} - \frac{9}{50} = \frac{27}{50}}\)

Do sprawdzenia:

Zad. 12. Dana jest funkcja gęstości pewnej zmiennej losowej X: \(\displaystyle{ f(x) = \begin{cases} 0 &\text{dla } x \epsilon(-\infty, 0> \cup (1, +\infty) \\3x^{2} &\text{dla } x\epsilon(0, 1>\end{cases}}\). Należy: wyznaczyć jej dystrybuantę, narysować wykres gęstości f i dystrybuanty F_x, wyznaczyć wartość oczekiwaną zmiennej losowej X, określić wartość wariancji i odchylenia standardowego tej zmiennej losowej oraz współczynnika zmienności i obliczyć modalną i medianę zmiennej losowej X.

- dystrybuanta:

\(\displaystyle{ dla...x\epsilon(-\infty, 0>}\)

\(\displaystyle{ \int\limits_{-\infty}^{x}f(t)dt = \int\limits_{-\infty}^{x}0dt = 0}\)

\(\displaystyle{ dla...x\epsilon(0, 1>}\)

\(\displaystyle{ \int\limits_{-\infty}^{x}f(t)dt = \int\limits_{-\infty}^{0}0dt + \int\limits_{0}^{x}3t^{2}dt = 3*\frac{1}{3}t^{3} = t^{3}\begin{tabular}{|rc|c}
x\\
\\
0\\
\end{tabular}}\)
\(\displaystyle{ = x^{3}-0^{3} = x^{3}}\)

\(\displaystyle{ dla...x\epsilon(1, +\infty)}\)

\(\displaystyle{ \int\limits_{-\infty}^{x}f(t)dt = \int\limits_{-\infty}^{0}0dt + \int\limits_{0}^{1}3t^{2}dt +
\int\limits_{1}^{x}0dt = t^{3}\begin{tabular}{|rc|c}
1\\
\\
0\\
\end{tabular}}\)
\(\displaystyle{ = 1^{3}-0^{3} = 1}\)

\(\displaystyle{ F(x) = \begin{cases} 0 &\text{dla } x \epsilon(-\infty, 0> \\x^{3} &\text{dla } x\epsilon(0, 1>\\1 &\text{dla } x\epsilon(1, +\infty>\\ \end{cases}}\)

- wykres gęstości:


- wykres dystrybuanty:


- wartość oczekiwana: \(\displaystyle{ EX = \int\limits_{-\infty}^{+\infty}xf(t)dx = \int\limits_{-\infty}^{0}x*0dx + \int\limits_{0}^{1}x*3x^{2}dx + \int\limits_{1}^{+\infty}x*0dx = \int\limits_{0}^{1}3x^{3}dx = 3*\frac{1}{4}x^{4} = \frac{3}{4}x^{4}\begin{tabular}{|rc|c}
1\\
\\
0\\
\end{tabular}}\)
\(\displaystyle{ = \frac{3}{4}*1^{4} - \frac{3}{4}*0^{4} = \frac{3}{4}}\)[/latex]
Ostatnio zmieniony 13 gru 2017, o 14:11 przez RagnarPL, łącznie zmieniany 23 razy.
janusz47
Użytkownik
Użytkownik
Posty: 7910
Rejestracja: 18 mar 2009, o 16:24
Płeć: Mężczyzna
Podziękował: 30 razy
Pomógł: 1670 razy

Podstawy statystyki: średnia, dominanta, kwartyl, wariancja

Post autor: janusz47 »

Proszę wykonać te same obliczenia parametrów opisu struktury dla chłopców. I w oparciu o ich interpretację, dokonać statystyczną analizę porównawczą populacji dziewcząt i chłopców.
RagnarPL
Użytkownik
Użytkownik
Posty: 3
Rejestracja: 10 gru 2017, o 14:07
Płeć: Mężczyzna
Lokalizacja: Kraków
Podziękował: 1 raz

Podstawy statystyki: średnia, dominanta, kwartyl, wariancja

Post autor: RagnarPL »

A zadanie drugie jest dobrze? I o co chodzi z tym rozkładem czasu trwania doświadczenia losowego?
janusz47
Użytkownik
Użytkownik
Posty: 7910
Rejestracja: 18 mar 2009, o 16:24
Płeć: Mężczyzna
Podziękował: 30 razy
Pomógł: 1670 razy

Podstawy statystyki: średnia, dominanta, kwartyl, wariancja

Post autor: janusz47 »

Jeśli w zadaniu 3 określimy doświadczenie losowe, polegające na kolejnym losowaniu bez zwracania kuli z urny, zawierającej pięć kul białych i jedną kulę czarną, to czasem trwania tego doświadczenia jest liczba losowań aż do wylosowania kuli czarnej.

Kula czarna za: pierwszym, drugim, ...,szóstym razem.
Awatar użytkownika
leg14
Użytkownik
Użytkownik
Posty: 3132
Rejestracja: 5 lis 2014, o 20:24
Płeć: Mężczyzna
Lokalizacja: Radom
Podziękował: 154 razy
Pomógł: 475 razy

Podstawy statystyki: średnia, dominanta, kwartyl, wariancja

Post autor: leg14 »

Czemu w pierwszym w wariancji w mianowniku masz \(\displaystyle{ 2}\) ?
Standardowy nieobciążony estymator wariancji miałby w tym wypadku \(\displaystyle{ 7}\) .
RagnarPL
Użytkownik
Użytkownik
Posty: 3
Rejestracja: 10 gru 2017, o 14:07
Płeć: Mężczyzna
Lokalizacja: Kraków
Podziękował: 1 raz

Re: Podstawy statystyki: średnia, dominanta, kwartyl, warian

Post autor: RagnarPL »

Dziękuję, będę próbował to zrobić.
leg14 - błąd przy wpisywaniu, już poprawione dzięki
ODPOWIEDZ